Spivak Exercice, Least Upper Bounds Chapter, Proof

In summary, the conversation is about understanding and proving a problem in Spicak Calculus regarding sets of numbers that are bounded above. The problem is to prove that the supremum of the sum of two nonempty sets is equal to the sum of the supremums of each individual set. The conversation includes a hint and a personal attempt at solving the problem, with a request for verification. The conversation ends with a confirmation that the solution seems correct.
  • #1
c.teixeira
42
0
Hi there!

I always think whether I am posting this correctly, or this belongs to the homework section. If so, my apologies.

I am trying to understand the solutions for a problem in Spicak Calculus, 3[itex]^{rd}[/itex] edition.

#8-13

The Problem:

"Let A and B bt two nonempty sets of numbers which are bounded above, and let A+B denote de set of all numbers x+y, with x in A and y in B. Prove that sup(A+B) = supA+supB. Hint: The inequality sup(A+B) ≤ sup A + sup B is easy. Why? To prove that supA + supB ≤ sup(A+B) it suffices to prove that supA + supB ≤ sup(A+B) + ε for all ε >0; begin by choosing x in A, and y in B with supA - x < [itex]\frac{ε}{2}[/itex] and supB -y <[itex]\frac{ε}{2}[/itex]."

Well, proving sup(A+B) ≤ supA + supB seems to be straightforward, but it is proving that supA + supB ≤ sup(A+B) that bothers me.

The explanation in the "Answers" chapter of the book, doesn't explain very clearly anything in this exercice. So, I have tried to prove it on my own:

1 - First of all, why is it sufficient to prove that supA + supB ≤ sup(A+B) + ε for all ε >0?

supA + supB - sup(A+B) ≤ ε, for all ε > 0.
Lets denote X = { ε : ε > 0}.
Then X is bounded below, and it is nonempty. The greatest lower bound is 0. And, since supA + supB - sup(A+B) is also a lower bound for X, we should have
supA + supB - sup(A+B) ≤ 0 ⇔ supA + supB ≤ sup(A+B).

2 - How do you prove that supA + supB ≤ sup(A+B) + ε for all ε >0?

If supA and supB are the greatest upper bounds for the A and B sets, respectly, then, for every ε > 0, there is an x in A , and an y in B such that:

→supA - x < [itex]\frac{ε}{2}[/itex];
→supB - y < [itex]\frac{ε}{2}[/itex];

→supA < [itex]\frac{ε}{2}[/itex] + x;
→supB < [itex]\frac{ε}{2}[/itex] + y;

Which means : supA + supB < ε + (x+y) ⇔ (x+y) > supA + supB - ε.. All this means, that for every ε > 0 , there is an x in A and y in B such that (x+y) > supA + supB - ε.

And finally, since sup(A+B)≥ x+y, for every x in A and y in B, we have for every ε > 0

sup(A+B) ≥ supA + supB -ε. Which, according to the first point is suffient.

Well, I feel pretty confident about this. However I would really appreciate if you could check it. I have spent way to much time around this one, and wanted to move on. ;D

Regards,

c.teixeira
 
Physics news on Phys.org
  • #2
I'm not sure I understand your first question, but if you show, in general, that:

x≤ y+1/n , for all n (subbing-in 1/n for ε ),

then it follows that x≤y

A way of showing this is that if x-y≤ 1/n for all n , then x≤y , by the

Archimedean Property. In a metric space,two elements cannot be

indefinitely-close ( using x-y as d(x,y) in ℝ ) , so we can conclude

x-y≤ 0 , so x≤y .
 
  • #3
Your solution seems fine.
 
  • #4
Sorry, I did not read your question carefully. I agree, it seems O.K.
 
  • #5


Hello c.teixeira,

It looks like you have a good understanding of the problem and have come up with a solid proof. Your reasoning is sound and you have correctly identified the steps needed to prove the statement. It is indeed sufficient to prove that supA + supB ≤ sup(A+B) + ε for all ε > 0, as you have shown in your explanation.

The only suggestion I have is to make sure to clearly state your assumptions and reasoning in your proof. For example, when you say "since sup(A+B) ≥ x+y, for every x in A and y in B," it would be helpful to state why this is true (i.e. because sup(A+B) is defined as the least upper bound for the set A+B). This will make your proof more clear and easier to follow.

Overall, great job on tackling this problem and coming up with a solid solution! Keep up the good work.
 

Related to Spivak Exercice, Least Upper Bounds Chapter, Proof

1. What is the concept of least upper bounds in mathematics?

The least upper bound of a set of numbers is the smallest number that is greater than or equal to all the numbers in the set. In simpler terms, it is the smallest possible upper bound for a set of numbers.

2. How does the concept of least upper bounds relate to the Spivak exercise in the Proof chapter?

In the Spivak exercise, you may be asked to prove that a set of numbers has a least upper bound. This involves using mathematical reasoning and concepts to show that the set has a smallest possible upper bound.

3. What is the importance of proving the existence of least upper bounds in mathematics?

The concept of least upper bounds is crucial in many mathematical proofs and theorems. It allows us to define precise limits and boundaries for sets of numbers, and helps us make conclusions about the behavior of these sets.

4. Are there any specific techniques or strategies for solving Spivak exercises related to least upper bounds?

There are various strategies that can be employed when solving Spivak exercises related to least upper bounds. These may include using proof by contradiction, induction, or direct proof techniques. It is important to carefully read and understand the problem before choosing a strategy to solve it.

5. Can the concept of least upper bounds be extended to sets of numbers in higher dimensions?

Yes, the concept of least upper bounds can be extended to sets of numbers in higher dimensions. For example, in three-dimensional space, the least upper bound of a set of points would be the smallest possible point that is greater than or equal to all the points in the set. This concept can also be extended to sets of numbers in even higher dimensions.

Similar threads

Replies
18
Views
2K
  • Calculus and Beyond Homework Help
Replies
2
Views
1K
  • Set Theory, Logic, Probability, Statistics
Replies
9
Views
2K
  • Precalculus Mathematics Homework Help
Replies
3
Views
6K
  • Topology and Analysis
Replies
4
Views
5K
  • Topology and Analysis
Replies
1
Views
2K
  • Topology and Analysis
Replies
8
Views
3K
Replies
2
Views
1K
Replies
1
Views
830
Replies
2
Views
2K
Back
Top